Stay ahead of learning milestones! Enroll in a class over the summer!

G
Topic
First Poster
Last Poster
Comics and triangles in perspective
srirampanchapakesan   0
14 minutes ago
Source: Own
Let a conic intersect the sides BC, CA, AB of triangle ABC at A1,A2,B1,B2,C1,C2.

T1 is the triangle formed by A1B2, B1C2, and C1A2.

T2 is the triangle formed by A2B1, B2C1 and C2A1.

Prove that the triangles ABC, T1 and T2 are pair-wise in perspective.

Also prove that all three centers of perspective coincide.
0 replies
srirampanchapakesan
14 minutes ago
0 replies
this geo is scarier than the omega variant
AwesomeYRY   10
N 16 minutes ago by GrantStar
Source: TSTST 2021/6
Triangles $ABC$ and $DEF$ share circumcircle $\Omega$ and incircle $\omega$ so that points $A,F,B,D,C,$ and $E$ occur in this order along $\Omega$. Let $\Delta_A$ be the triangle formed by lines $AB,AC,$ and $EF,$ and define triangles $\Delta_B, \Delta_C, \ldots, \Delta_F$ similarly. Furthermore, let $\Omega_A$ and $\omega_A$ be the circumcircle and incircle of triangle $\Delta_A$, respectively, and define circles $\Omega_B, \omega_B, \ldots, \Omega_F, \omega_F$ similarly.

(a) Prove that the two common external tangents to circles $\Omega_A$ and $\Omega_D$ and the two common external tangents to $\omega_A$ and $\omega_D$ are either concurrent or pairwise parallel.

(b) Suppose that these four lines meet at point $T_A$, and define points $T_B$ and $T_C$ similarly. Prove that points $T_A,T_B$, and $T_C$ are collinear.

Nikolai Beluhov
10 replies
1 viewing
AwesomeYRY
Dec 13, 2021
GrantStar
16 minutes ago
3-var inequality
sqing   1
N 21 minutes ago by sqing
Source: Own
Let $ a,b,c\geq 0 ,a+b+c =4. $ Prove that
$$a +ab^2 +ab^2c \leq\frac{33}{4}+2\sqrt 2$$$$a +ab^2 +abc \leq \frac{2(100+13\sqrt {13})}{27}$$$$a +a^2b + a b^2c^3\leq \frac{2(82+19\sqrt {19})}{27}$$
1 reply
sqing
37 minutes ago
sqing
21 minutes ago
Hard to approach it !
BogG   128
N 26 minutes ago by alexanderchew
Source: Swiss Imo Selection 2006
Let $\triangle ABC$ be an acute-angled triangle with $AB \not= AC$. Let $H$ be the orthocenter of triangle $ABC$, and let $M$ be the midpoint of the side $BC$. Let $D$ be a point on the side $AB$ and $E$ a point on the side $AC$ such that $AE=AD$ and the points $D$, $H$, $E$ are on the same line. Prove that the line $HM$ is perpendicular to the common chord of the circumscribed circles of triangle $\triangle ABC$ and triangle $\triangle ADE$.
128 replies
BogG
May 25, 2006
alexanderchew
26 minutes ago
A nice and easy gem off of StackExchange
NamelyOrange   1
N 27 minutes ago by NamelyOrange
Source: https://math.stackexchange.com/questions/3818796/
Define $S$ as the set of all numbers of the form $2^i5^j$ for some nonnegative $i$ and $j$. Find (with proof) all pairs $(m,n)$ such that $m,n\in S$ and $m-n=1$.


Rephrased: Solve $2^a5^b-2^c5^d=1$ over $(\mathbb{N}_0)^4$, and prove that your solution(s) is/are the only one(s).
1 reply
NamelyOrange
Yesterday at 8:13 PM
NamelyOrange
27 minutes ago
Popular children at camp with algebra and geometry
Assassino9931   1
N 29 minutes ago by internationalnick123456
Source: RMM Shortlist 2024 C3
Fix an odd integer $n\geq 3$. At a maths camp, there are $n^2$ children, each of whom selects
either algebra or geometry as their favourite topic. At lunch, they sit at $n$ tables, with $n$ children
on each table, and start talking about mathematics. A child is said to be popular if their favourite
topic has a majority at their table. For dinner, the students again sit at $n$ tables, with $n$ children
on each table, such that no two children share a table at both lunch and dinner. Determine the
minimal number of young mathematicians who are popular at both mealtimes. (The minimum is across all sets of topic preferences and seating arrangements.)
1 reply
Assassino9931
5 hours ago
internationalnick123456
29 minutes ago
3-var inequality
sqing   0
an hour ago
Source: Own
Let $ a,b,c\geq 0 ,a+b+c =1. $ Prove that
$$\frac{ab}{2c+1} +\frac{bc}{2a+1} +\frac{ca}{2b+1}+\frac{27}{20} abc\leq \frac{1}{4} $$
0 replies
sqing
an hour ago
0 replies
Inspired by giangtruong13
sqing   3
N an hour ago by sqing
Source: Own
Let $ a,b \geq 0 $ and $a^3-b^3=2 $.Prove that $$ a^2-ab+b^2 \geq   \sqrt[3]{2}  $$Let $ a,b \geq 0 $ and $a^3+b^3=2 $.Prove that $$ 3\geq a^2+ab+b^2 \geq   \sqrt[3]{4}  $$
3 replies
sqing
2 hours ago
sqing
an hour ago
Very easy case of a folklore polynomial equation
Assassino9931   2
N an hour ago by luutrongphuc
Source: Bulgaria EGMO TST 2025 P6
Determine all polynomials $P(x)$ of odd degree with real coefficients such that $P(x^2 + 2025) = P(x)^2 + 2025$.
2 replies
Assassino9931
5 hours ago
luutrongphuc
an hour ago
Something nice
KhuongTrang   30
N 2 hours ago by KhuongTrang
Source: own
Problem. Given $a,b,c$ be non-negative real numbers such that $ab+bc+ca=1.$ Prove that

$$\sqrt{a+1}+\sqrt{b+1}+\sqrt{c+1}\le 1+2\sqrt{a+b+c+abc}.$$
30 replies
KhuongTrang
Nov 1, 2023
KhuongTrang
2 hours ago
Tangency geo
Assassino9931   1
N 2 hours ago by sami1618
Source: RMM Shortlist 2024 G1
Let $ABC$ be an acute triangle with $\angle ABC > 45^{\circ}$ and $\angle ACB > 45^{\circ}$. Let $M$ be the midpoint of the side $BC$. The circumcircle of triangle $ABM$ intersects the side $AC$ again at $X\neq A$ and the circumcircle of triangle $ACM$ intersects the side $AB$ again at $Y\neq A$. The point $P$ lies on the perpendicular bisector of the segment $BC$ so that the points $P$ and $A$ lie on the same side of $XY$ and $\angle XPY = 90^{\circ} + \angle BAC$. Prove that the circumcircles of triangles $BPY$ and $CPX$ are tangent.
1 reply
Assassino9931
6 hours ago
sami1618
2 hours ago
Removing cell to tile with L tetromino
ItzsleepyXD   1
N 2 hours ago by internationalnick123456
Source: [not own] , Mock Thailand Mathematic Olympiad P4
Consider $2025\times 2025$ Define a cell with $\textit{Nice}$ property if after remove that cell from the board The board can be tile with $L$ tetromino.
Find the number of position of $\textit{Nice}$ cell $\newline$ Note: $L$ tetromino can be rotated but not flipped
1 reply
ItzsleepyXD
Apr 30, 2025
internationalnick123456
2 hours ago
IMO Problem 4
iandrei   105
N Apr 13, 2025 by cj13609517288
Source: IMO ShortList 2003, geometry problem 1
Let $ABCD$ be a cyclic quadrilateral. Let $P$, $Q$, $R$ be the feet of the perpendiculars from $D$ to the lines $BC$, $CA$, $AB$, respectively. Show that $PQ=QR$ if and only if the bisectors of $\angle ABC$ and $\angle ADC$ are concurrent with $AC$.
105 replies
iandrei
Jul 14, 2003
cj13609517288
Apr 13, 2025
Source: IMO ShortList 2003, geometry problem 1
The post below has been deleted. Click to close.
This post has been deleted. Click here to see post.
HamstPan38825
8857 posts
#95
Y by
here's a fourth projective solution to this i guess

Note that $\overline{PQR}$ is the Simson line of $D$ with respect to triangle $ABC$. Hence for $E = \overline{DP} \cap (ABCD)$, it is parallel to $\overline{AE}$. Then $$(AC;BD) \stackrel P=(FB;EC) \stackrel A= (PR; Q\infty_{PQR})$$which implies the result.
Z K Y
The post below has been deleted. Click to close.
This post has been deleted. Click here to see post.
MagicalToaster53
159 posts
#96
Y by
Notice that the concurrent angle condition is identical to the condition that $ABCD$ is a harmonic quadrilateral. Observe now that \[\frac{PQ}{QD} = \frac{\sin \angle PDQ}{\sin \angle QPD} = \frac{\sin \angle BAC}{\sin \angle DAC},\]and \[\frac{QR}{QD} = \frac{\sin \angle QDR}{\sin \angle DRQ} = \frac{\sin \angle ACB}{\sin \angle DBA}. \]Hence, \[\frac{PQ}{QR} = \frac{\sin \angle BAC}{\sin \angle DAC} \cdot \frac{\sin \angle DBA}{\sin \angle ACB} = \frac{BC}{BA} \cdot \frac{DA}{DC} = 1 \iff ABCD \text{ is harmonic,}\]as desired. $\blacksquare$
Z K Y
The post below has been deleted. Click to close.
This post has been deleted. Click here to see post.
Markas
105 posts
#97 • 1 Y
Y by panche
If the bisectors meet on AC this gives us $\frac{AD}{DC} = \frac{AB}{BC}$. From ABCD cyclic $\angle BAC = \angle PCD$, so $\triangle CPD \sim \triangle ARD$, so $\frac{AR}{CP} = \frac{AD}{DC}$. From Menelaus on $\triangle BPR$ we get $\frac{BA}{AR}. \frac{RQ}{QP}. \frac{PC}{CB} = 1$ (we can do this since P, Q, R lie on one line aka Simpson line) and $\frac{BA}{AR}. \frac{PC}{CB} = \frac{AD}{DC}. \frac{DC}{AD} = 1$. This means $\frac{PQ}{QP} = 1$ $\Rightarrow$ PQ = QP. In the other direction is the same because the pair of similar triangles is there, so we get that $\frac{BA}{CB} = \frac{AD}{DC}$ which is equivalent to the fact that the bisectors meet on AC. We are ready.
Z K Y
The post below has been deleted. Click to close.
This post has been deleted. Click here to see post.
BestAOPS
707 posts
#98 • 1 Y
Y by teomihai
Notice that $P,Q,R$ lie on the Simson line.

Let $X$ be the second intersection of line $DQ$ with the circle $\omega$ circumscribing $ABCD$. We claim that $\overline{BX} \parallel \overline{RQ}$. Using directed angles,
\[ \angle BXD = \angle BAD = \angle RAD = \angle RQD, \]and thus, $\overline{BX}$ is parallel to the Simson line.

Next, note that by the angle bisector theorem, the bisector condition is equivalent to $ABCD$ being harmonic. Additionally, we have
\[ (A,C;B,D) \stackrel{X}{=} (A,C;\overline{BX} \cap \overline{AC}, Q) \stackrel{B}{=} (R,P;P_\infty,Q), \]and since $(R,P;P_\infty,Q) = -1 \iff PQ=QR$, we are done.
Z K Y
The post below has been deleted. Click to close.
This post has been deleted. Click here to see post.
ezpotd
1262 posts
#99 • 1 Y
Y by m4thbl3nd3r
Complex bash, let $A = w, C = \overline{w}, B = b, D = d$. Let $M,N = 1,-1$ be the arc midpoints of $AC$. Note $BD,MN$ meet a point whose polar is parallel to $AC$. If $BM,DN,AC$ are concurrent (or $BN, DM, AC$), then Brokard tells us that $AC$ is precisely this polar, so the condition can be reduces to the intersection of tangents from $A,C$ being the intersection of $BD, MN$. The intersection of $BD, MN$ is just $\frac{b + d}{bd + 1}$, so the desired condition is just $\frac{b + d}{bd + 1} = \frac{2}{w + \overline{w}}$, rearrange to get $2bd + 2 = (b + d)(w + \overline{w})$. Now for the condition that $PQ = QR$. We dilate by two and instead find a condition for $P'Q' = Q'R'$. By Simson line, it just suffices to show $p + r = 2q$. The reflection of $D$ over $AB$ is just $w + b - \frac{bw}{d}$, likewise the reflection of $D$ over $BC$ is just $b + \overline{w} - \frac{b\overline{w}}{d}$. The reflection of $D$ over $AC$ is just $w + \overline w - \frac 1d$. Now the condition is $2b + w + \overline{w} - \frac bd (w + \overline{w}) = 2(w + \overline{w} ) - \frac 2d$. Multiplying by $d$, an equivalent condition is $2bd + (d -b)(w + \overline{w}) = 2d(w + \overline{w}) - 2$, rearranging we get $(b + d)(w + \overline{w}) = 2bd + 2$, as desired. Since the conditions are equivalent, we can conclude that $PQ = QR$ and the concurrency happen at exactly the same configurations, we are done.
Z K Y
The post below has been deleted. Click to close.
This post has been deleted. Click here to see post.
peppapig_
281 posts
#100
Y by
First, note that if the angle bisectors of $\angle ABC$ and $\angle ADC$ meet on segment $AC$, we then have
\[\frac{AB}{CB}=\frac{AD}{CD} \iff AB*CD=AD*BC \iff (AC;BD)=-1,\]and by the uniqueness of the harmonic conjugate, this means that $B$ is the unique point on $(ACD)$ such that $(AC;BD)=-1$. By the harmonic quadrilateral configuration, this means that $BD$ is the $D$-symmedian of $\triangle ACD$. It then suffices to show that
\[PQ=QR\iff BD\text{ is the } D\text{-symmedian of }\triangle ACD.\]
To start, we make the following claims.

***

Claim 1. $P$, $Q$, $R$ are collinear.

Proof.
This is immediate from the Simson Line at $D$ with respect to $\triangle ABC$.

***

Claim 2. There exists a spiral similarity centered at $D$ sending $\triangle DRP$ to $\triangle DAC$.

Proof.
All angles below are directed.

We can prove this through (directed) angle chasing. First, note that since $\angle CQD=\angle CPD=90$, we have that $C$, $P$, $D$, $Q$ are concyclic, which gives us that
\[\angle RPD=\angle QPD=\angle QCD=\angle ACD,\]and since $\angle ARD=\angle AQD=90$, we have that $A$, $R$, $Q$, $D$ concyclic, meaning that
\[\angle PRD=\angle QRD=\angle QAD=\angle CAD,\]and these two combined gives us that $\triangle DRP\sim \triangle DAC$, as desired. This proves our claim.

***

Now, let $M$ be the midpoint of $AC$. Notice that $Q$ is the midpoint of $RP$ if and only if the spiral similarity described in Claim 2. sends $Q$ to $M$. However, note that
\[\angle Q\rightarrow M \iff \angle MDQ=\angle CDP \iff 90-\angle MDQ=90-\angle CDP \iff \angle DMQ=\angle DCP,\]and since $\angle DCP=\angle BAD$ by cyclic properties, and $\angle DMQ=\angle DMC$, this gives us that
\[\iff \angle BAD=\angle DMC.\]
Now, let us fix $A$, $C$, and $D$. We make the following claim.

***

Claim 3. If $\angle BAD=\angle DMC$, then $B\neq D$ must be the unique point on $(ACD)$ such that $BD$ is the $D$-symmedian of $\triangle ACD$.

Proof.
Notice that since $\angle DMC$ is fixed, $\angle BAD$ must also be fixed, meaning that there is a unique point on $(ACD)$ such that $\angle BAD=\angle DMC$. Now, we can angle chase. We get that
\[\angle ADM=180-\angle AMD-\angle MAD=\angle CMD-\angle MAD=\angle BAD-\angle MAD=\angle BAC=\angle BDC,\]which means that $BD$ is indeed the $D$-symmedian of $\triangle ABC$, as desired. This proves our claim.

***

By Claim 3. means that $\angle BAD=\angle DMC$ is equivalent to the condition that $BD$ is the $D$-symmedian of $ADC$. However, we also established earlier that the former was equivalent to $Q$ being the midpoint of $PR$, meaning that
\[PQ=QR\iff BD\text{ is the } D\text{-symmedian of }\triangle ACD,\]as desired. Since
\[BD\text{ is the } D\text{-symmedian of }\triangle ACD\iff (AC;BD)=-1\iff \text{the angle bisector of }\angle ABC \text{ and } \angle ADC \text{ meet on } AC,\]we have that the angle bisectors meet on $AC$ if and only if $PQ=QR$, which is what we wished to prove. This completes our proof.
This post has been edited 1 time. Last edited by peppapig_, Nov 1, 2024, 10:11 PM
Reason: \implies but really \iff
Z K Y
The post below has been deleted. Click to close.
This post has been deleted. Click here to see post.
shendrew7
794 posts
#101
Y by
Notice that
\[PQ = QR \iff AD \sin \angle A = CD \sin \angle C \iff AD \cdot BC = CD \cdot AB. \quad \blacksquare\]
Z K Y
The post below has been deleted. Click to close.
This post has been deleted. Click here to see post.
lnzhonglp
120 posts
#102
Y by
Let $DP$ intersect $(ABC)$ again at $K$. By Simson line $P, Q, R$ are collinear, and $\measuredangle QPD = \measuredangle PCD = \measuredangle AKD$ so $AK \parallel PQ$. Then $$(AC;BD) \overset{P}{=} (AP \cap (ABC), B; C, K) \overset{A}{=} (PR; Q\infty),$$which concludes by angle bisector theorem.
Z K Y
The post below has been deleted. Click to close.
This post has been deleted. Click here to see post.
DakshAggarwalRedsurgance
10 posts
#103
Y by
arshakus wrote:
hey guys)
I solved this problem in about 10 minutes, but I think there is some thing wrong because I didn't use the fact that $ABCD$ is cyclic.
Can it be so?

Yes. (same here) it is not a requirement at all . The fact that it is cyclic helps in the creation of simson line and just makes the diagram easy to construct plus it gets rid of nasty configuration issues. I was able to see some possible issues(which were not quite problematic are so) But there could have been some config that I missed. I asked a few people too and all of them agree. IMO committee wanted the problem to own its own merit instead of relying on configs so yippee
Z K Y
The post below has been deleted. Click to close.
This post has been deleted. Click here to see post.
Perelman1000000
110 posts
#104
Y by
$\boxed{nice problem}$
Attachments:
Z K Y
The post below has been deleted. Click to close.
This post has been deleted. Click here to see post.
Mquej555
15 posts
#105
Y by
How can it be to IMO?

\[PQ = QR \iff AD \sin \angle A = CD \sin \angle C \iff AD \cdot BC = CD \cdot AB. \quad \blacksquare\]
Z K Y
The post below has been deleted. Click to close.
This post has been deleted. Click here to see post.
LeYohan
41 posts
#106
Y by
My first IMO #4 sol :blush:

We notice that second condition is equivalent to showing that $ABCD$ is an harmonic quadrilateral because of the angle bisector theorem, so we want to proof that $\frac{CB}{AB}=\frac{CD}{AD}$.

Assuming that $PQ=QR$, it's well known that $P-Q-R$, so we proceed by applying Menelaus's theorem on $\triangle PRB$ with transversal line $AC$, getting that $\frac{CB}{AB}=\frac{CP}{AR}$, so now we want to show that $\frac{CP}{AR}=\frac{CD}{AD}$, but just notice that $\triangle DPC \sim \triangle DRA$ which gives the proportion.

Assuming that $\frac{CB}{AB}=\frac{CD}{AD}$, by the previous proportion we know $\frac{CP}{AR}=\frac{CD}{AD}$ so by applying Menelaus's theorem again on $\triangle PRB$ with transversal line $AC$, we obtain $PQ=QR$, so we're done. $\square$
This post has been edited 1 time. Last edited by LeYohan, Feb 10, 2025, 10:12 PM
Z K Y
The post below has been deleted. Click to close.
This post has been deleted. Click here to see post.
Maximilian113
574 posts
#107
Y by
By the Angle Bisector Theorem, the second condition is equivalent to $$\frac{AB}{BC} = \frac{AD}{CD}.$$
Now, note that $DQAR$ is a cyclic quadrilateral as $\angle DQA = \angle DRA = 90^\circ.$ Therefore by the Law of Sines $$\frac{QR}{\sin \angle QDR} = \frac{DR}{\sin \angle DQR} \iff \frac{QR}{\sin \angle BAC} = \frac{DR}{\sin \angle DAC}.$$Similarly $$\frac{PQ}{\sin \angle BCA} = \frac{DR}{\sin \angle DCA}.$$Dividing these equations, it follows that $$PQ=QR \iff \frac{\sin \angle BAC}{\sin \angle BCA} = \frac{\sin \angle DAC}{\sin \angle DCA} \iff \frac{AB}{BC} = \frac{AD}{CD}$$by the Law of Sines. QED
Z K Y
The post below has been deleted. Click to close.
This post has been deleted. Click here to see post.
cj13609517288
1900 posts
#108
Y by
The first condition, after multiplying by $2$, subtracting from $a+b+c+d$, then multiplying by $d$, is equivalent to
\[2ac+2bd=ab+bc+cd+da\]in complex. The second condition is equivalent to $ABCD$ being harmonic, which is
\[(a-b)(c-d)=-(c-b)(a-d)\Longleftrightarrow 2ac+2bc=ab+bc+cd+da.\]Wow. $\blacksquare$
Z K Y
The post below has been deleted. Click to close.
This post has been deleted. Click here to see post.
cj13609517288
1900 posts
#109 • 1 Y
Y by Maximilian113
Okay fine here's a synthetic solution.

Let's prove the forwards direction. Let $B'$ be on $(ABCD)$ such that $BB'\parallel AC$, and similarly for $D'$. We have
\[
-1=(P,R;Q,\infty_{PR})
\stackrel{D}{=}(\infty_{\perp BA},\infty_{\perp BC};\infty_{\perp AC},\infty_{PR})
\stackrel{\text{rotate }90^\circ}{=}(\infty_{BA},\infty_{BC};\infty_{AC},\infty_{\perp PR})
\stackrel{B}{=}(A,C;B',B\infty_{\perp PR}\cap(ABCD)).
\]But we claim that the unique such point $B\infty_{\perp PR}\cap(ABCD)$ is exactly $D'$. Indeed, $BD$ is the $B$-circumcenter cevian of triangle $BPR$ (since $D$ is the antipode), so $BD'$, being isogonal to $BD$, is perpendicular to $PR$.

Thus $(A,C;B,D)=-1$ and the angle bisector thing follows by the angle bisector theorem. Everything in this proof was reversible so we get the other direction too. $\blacksquare$
Z K Y
N Quick Reply
G
H
=
a